Thứ Sáu, 6 tháng 11, 2015

Tổng hợp một số bài bất đẳng thức:
                                                                                          Nguồn: VMF

Bài 1:
Cho abc=1 va $a^{3}> 36.  CMR  :\frac{a^{2}}{3}+b^{2}+c^{2}> ab +bc+ca$}
Lời giải:
$VT-VP=\frac{a^{2}}{4}+b^{2}+c^{2}-ab-bc+2bc+\frac{a^{2}}{12}=(\frac{a}{2}-b-c)^{2}+\frac{a^{2}-36bc}{12}>0\Rightarrow$ đpcm
Cách khác:
Từ giả thiết suy ra $a>0$ và $bc>0$. Bất đẳng thức cần chứng minh tương đương với
\[\dfrac{a^2}{3}+(b+c)^2-3bc-a(b+c)\ge 0\\ \iff \dfrac{1}{3}+\left(\dfrac{b+c}{a}\right)^2-\dfrac{b+c}{a}-\dfrac{3}{a^3}\ge 0\]
Vì $a^3>36$ nên \[\dfrac{1}{3}+\left(\dfrac{b+c}{a}\right)^2-\dfrac{b+c}{a}-\dfrac{3}{a^3}> \left(\dfrac{b+c}{a}\right)^2-\dfrac{b+c}{a}+ \dfrac{1}{4}= \left(\dfrac{b+c}{a}-\dfrac{1}{2}\right)^2 >0\]

Bài 2:
Với a,b,c >0; n ∈ N*.CMR:
$\frac{a^{n}}{b+c}+\frac{b^{n}}{a+c}+\frac{c^{n}}{a+b}\geq \frac{3}{2}\left ( \frac{a^{n}+b^{n}+c^{n}}{a+b+c} \right )$
Lời giải:
$\sum \frac{a^{n}}{b+c}\geq \frac{1}{3}(\sum a^{n})(\sum \frac{1}{a+b})\geq \frac{1}{3}(\sum a^{n})(\frac{9}{2(a+b+c)})=\frac{3}{2}(\frac{\sum a^{n}}{\sum a})$

Bài 3:
Cho $x,y,z >0$ thỏa điều kiện $x^{2}+y^{2}+z^{2}=9$
Tìm giá trị nhỏ nhất của $P=\frac{x^5}{y^2}+\frac{y^5}{z^2}+\frac{z^5}{x^2}$
Lời giải:
Theo $Cauchy$ Ta có:
$$\dfrac{x^5}{y^2}+\dfrac{x^5}{y^2}+\sqrt{3}y^2+\sqrt{3}y^2+3\sqrt{3}\ge \sqrt{3}x^2$$
Cách khác:
Sử dụng Cauchy-Schwarzt ta có 
       $\frac{x^5}{y^2}+\frac{y^5}{z^2}+\frac{z^5}{x^2}\geqslant \frac{(x^3+y^3+z^3)^2}{xy^2+yz^2+zx^2}$
Sử dụng Cauchy-Schwarzt và AM-GM ta có 
       $xy^2+yz^2+zx^2\leqslant \sqrt{(x^2+y^2+z^2)(x^2y^2+y^2z^2+z^2x^2)}\leqslant \sqrt{\frac{(x^2+y^2+z^2)^3}{3}}=3$
Do đó $P\geqslant \frac{(x^3+y^3+z^3)^2}{3}\geqslant \frac{(x^2+y^2+z^2)^3}{9}=3$ 
Đẳng thức xảy ra khi $x=y=z=1$

Bài 4:
Ch0 $a>0$ và $n$ là 1 số tự nhiên
Chứng minh rằng $a^n+\frac{1}{a^n}-2\geqslant n^2(a+\frac{1}{a}-2)$
Lời giải:
Bất đẳng thức tương đương với $(a^{n-1}+a^{n-2}+...+a+1)\geq n^2a^{n-1}$ (hiển nhiên theo AM-GM)
Cách khác:
Do tính đối xứng giữa a và $\frac{1}{a}$ nên ta có thể giả sử a ≥ 1.  đặt $\sqrt{a}$ =x ≥ 1.bdt $\Leftrightarrow$ $x^{2n}+\frac{1}{x^{2n}}-2 \geq n^{2}(x^{2}+\frac{1}{x^{2}}-2)\Leftrightarrow (x^{n}-\frac{1}{x^{n}})^{2}\geq n^{2}(x-\frac{1}{x})^{2} \Leftrightarrow $x^{n}-\frac{1}{x^{n}}\geq n(x-\frac{1}{x})$①.
Với x=1 thì ① đúng
Với x>1 thì ① $\Leftrightarrow x^{n-1} +x^{n-3} ...+\frac{1}{x^{n-3}}+\frac{1}{x^{n-1}}\geq n$ (đúng vì theo bđt AM-GM).
Dấu bằng xảy ra khi x=1 $\Leftrightarrow a=1$

Bài 5:
Cho $a,b,c,d$ là các số thực thỏa mãn $\left\{\begin{matrix} a+b+c+d=0\\a^2+b^2+c^2+d^2=2 \end{matrix}\right.$
Tìm GTLN của $P=abcd$
Lời giải:
Áp dụng AM-GM ta có 
$2=\sum a^{2}\geq 4\sqrt[4]{\prod a^{2}}\Rightarrow \sqrt{\left | abcd \right |}\leq \frac{1}{2}\Rightarrow abcd\leq \frac{1}{4}$
Dấu bằng xảy ra khi $a=b=-c=-d=\frac{1}{\sqrt{2}}$ và các hoán vị của chúng

Bài 6:
Cho $a,\,b,\,c\geq 0$ thỏa mãn $a+b+c=1.$ Tìm giá trị lớn nhất của biểu thức: $$P=abc\left(a^2+b^2+c^2\right)$$
Lời giải:
Ta có: $P=abc(a+b+c)(a^{2}+b^{2}+c^{2})\leq \frac{1}{3}(ab+bc+ca)^{2}(a^{2}+b^{2}+c^{2})$

Mặt khác, lại có: $(ab+bc+ca)^{2}(a^{2}+b^{2}+c^{2})\leq \left (\frac{(a+b+c)^{2}}{3} \right )^{3}=\frac{1}{27}$

Do đó: $P\leq \frac{1}{81}$

Dấu bằng xảy ra khi và chỉ khi $a=b=c= \frac{1}{3}$

Bài 7:
Cho các số thực $x,\,y>0$ thỏa mãn $3x+y\leq1.$ Tìm giá trị nhỏ nhất của biểu thức: $$S=\dfrac{1}{x}+\dfrac{1}{\sqrt{xy}}$$
Lời giải:
$S\geq \frac{1}{x}+\frac{1}{\sqrt{x(1-3x)}}$
$\geq \frac{1}{x}+\frac{2}{1-2x}=\frac{2}{x(1-x)}\geq \frac{8}{(x+1-x)^{2}}=8$
Dấu "=" xảy ra $\Leftrightarrow x=\frac{1}{4}$

Bài 8:
Cho các số thực a,b,c,x,y thỏa mãn $$ax-by=\sqrt{3}$$ .
Tìm GTNN của $F= a^{2}+b^{2}+x^{2}+y^{2}+ bx +ay$
Lời giải:
Sử dụng giả thiết $ax-by=\sqrt{3}$ ta có:
$$(a^2+b^2)(x^2+y^2)=(ax+by)^2+(ax-by)^2=(ax+by)^2+3$$
Áp dụng bất đẳng thức $Cauchy$ , suy ra:
$$a^2+b^2=x^2+y^2=(a^2+b^2)+(x^2+y^2) \\ \ge 2\sqrt{(a^2+b^2)(x^2+y^2)}=2\sqrt{(ax+by)^2+3}$$
Do đó, ta đưa về bài toán tìm GTNN của: $2\sqrt{x^2+3}+x$ trong đó $x=ax+by$
Ta có:
$$\left(2\sqrt{x^2+3}+x\right)^2=4(x^2+3)+4x\sqrt{x^2+3}+x^2 \\ = (x^2+3)+4x\sqrt{x^2+3}+4x^2+9 \\ = \left(\sqrt{x^2+3}+2x\right)^2+9\ge 9$$
$$\Rightarrow 2\sqrt{x^2+3}+x\ge 3$$
Vậy $\text{MinT}=\fbox{3}$


Bài 9: (An Nguyễn)
Cho $a, b, c> 0$. CMR:
$\dfrac{a^3}{a^2+ab+b^2}+\dfrac{b^3}{b^2+bc+c^2}+\dfrac{c^3}{a^2+ac+c^2}\geq\dfrac{a+b+c}{3}$
Lời giải:
Áp dụng hệ quả của bất đẳng thức Bu-nhi-a-cốp-xki ta có:
$\dfrac{a^{3}}{a^{2}+ab+b^{2}}+\dfrac{b^{3}}{b^{2}+bc+c^{2}}+\dfrac{c^{3}}{c^{2}+ca+a^{2}}\geq \dfrac{(a^{2}+b^{2}+c^{2})^{2}}{(a^{2}+b^{2}+c^{2})(a+b+c)}=\dfrac{a^{2}+b^{2}+c^{2}}{a+b+c}\geq \dfrac{a+b+c}{3}$(đpcm)
Dấu "=" xảy ra$\Leftrightarrow a=b=c$

Thứ Năm, 5 tháng 11, 2015

MỘT SỐ PHƯƠNG PHÁP GIẢI TOÁN PHƯƠNG TRÌNH

Trong kiến thức toán THPT, các dạng bài tập về phương trình rất phong phú, đa dạng. Hầu hết các đề thi đại thường chốt lại bằng một bài phương trình hoặc hệ phương trình, do vậy có thể nói dạng toán này khá quan trọng. Chính vì thế, hôm nay mình mở topic này để cùng mọi người tham gia và trao đổi kinh nghiệm học tập về dạng này, rất mong mọi người quan tâm để cùng nhau phát triển diễn đàn VHF của chúng ta ngày 1 hoàn thiện. Xin cảm ơn! :) :like:
Chúng ta đã biết một số dạng như: phương trình vô tỉ, phương trình đẳng cấp, hệ đối xứng - nửa đối xứng, phương trình có một nghiệm duy nhất, hệ phương trình đẳng cấp, hệ phương trình có 1 phương trình tích...Do đó cũng có rất nhiều cách giải khác nhau như:
-  Phương pháp nhẩm nghiệm (bằng máy tính)
-  Phương pháp đặt ẩn phụ
-  Phương pháp đổi biến
-  Phương pháp nhân liên hợp
-  Phương pháp sử dụng bất đẳng thức
...
Một số phương pháp giải:
*  Phương pháp đổi biến:
I) Phương trình đẳng cấp: $aP(x) + bQ(x) = c\sqrt{P(x).Q(x)}$    (1)
Phương pháp: Đặt $u = \sqrt{P(x)}$, $v = \sqrt{Q(x)}$       (u, v >= 0)
                  $(1) \Leftrightarrow  au^{2} + bv^{2} = c.u.v$    (2)
                  Nhận xét: v = 0 là nghiệm của (2) ?
                  $(2) \Leftrightarrow  a(\dfrac{u}{v})^{2} +  b - c.\dfrac{u}{v} = 0$
* Phương pháp nhân liên hợp:
II) Nghiệm vô tỉ
Sử dụng: $(\sqrt{a} - \sqrt{b})(\sqrt{a} + \sqrt{b}) = a - b$
            $(\sqrt[3]{a} \underline{+} \sqrt[3]{b})(\sqrt[3]{a^{2}} \overline{+} \sqrt[3]{ab} + \sqrt[3]{b^{2}}) = a \underline{+} b$
*  Dạng phương trình: $\sqrt[3]{A(x)} \pm  \sqrt[3]{B(x)} = \sqrt[3]{C(x)}$
Phương pháp: Lập phương hai vế.
* Dạng phương trình: $a^{2} + bx + c = \sqrt{px^{2} + qx + r} (a.p \neq  0)$
Phương pháp:
         TH1: $\dfrac{a}{p} = \dfrac{b}{q}$. Đặt $t = \sqrt{px^{2} + qx + r}$.
                Đưa về dạng phương trình bậc 2.
         TH2: $\left\{\begin{matrix} p = -b & & & \\ q = \dfrac{1 - b^{2}}{a} & & & \\ r = \dfrac{-c(1 + b)}{a} & & & \end{matrix}\right.$
                Đặt $t = ax^{2} + bx + c$ rồi đưa về hệ đối xứng loại 2
Phương pháp nâng lên luỹ thừa 2 vế:
Dạng 1: $\sqrt{f(x)}=\sqrt{g(x)}\Leftrightarrow \left\{\begin{matrix}
&f(x)=g(x) \\
&f(x)\geq 0(hoặc g(x)\geq 0)
\end{matrix}\right.$
VD: Giải phương trình: $\sqrt{x^{2}+2x+4}=\sqrt{2-x}$(1)
(1)$\Leftrightarrow \left\{\begin{matrix}
&x^{2}+2x+4=2-x \\
&x\leq 2
\end{matrix}\right.$
$\Leftrightarrow \left\{\begin{matrix}
&x^{2}+3x+2=0 \\
&x\leq 2
\end{matrix}\right.$
$\Leftrightarrow x=-1$(thoả mãn) hoặc $x=-2$(thoả mãn)
Dạng 2: $\sqrt{f(x)}=g(x)\Leftrightarrow \left\{\begin{matrix}
&g(x)\geq 0 \\
&f(x)=g(x)^{2}
\end{matrix}\right.$
VD: Giải phương trình: $\sqrt{4+2x-x^{2}}=x-2$(2)
(2)$\Leftrightarrow \left\{\begin{matrix}
&x\geq 2 \\
&4+2x-x^{2}=x-2
\end{matrix}\right.$
$\Leftrightarrow \left\{\begin{matrix}
&x\geq 2 \\
&x^{2}-x-6=0
\end{matrix}\right.$
$\Leftrightarrow x=-2$(không thoả mãn) hoặc $x=3$(thoả mãn)
Dạng 3: $\sqrt{f(x)}-\sqrt{g(x)}=\sqrt{h(x)}\Leftrightarrow \sqrt{f(x)}=\sqrt{g(x)}+\sqrt{h(x)}$
$\Leftrightarrow \left\{\begin{matrix}
&g(x)\geq 0  & \\
&h(x)\geq 0  & \\
&f(x)=g(x)+h(x)+2\sqrt{g(x).h(x)}  &
\end{matrix}\right.$
VD: Giải phương trình: $\sqrt{3x+1}-\sqrt{x+4}=1$(3)
(3)$\Leftrightarrow \sqrt{3x+1}=1+\sqrt{x+4}$
$\Leftrightarrow \left\{\begin{matrix}
&x\geq -4 \\
&3x+1=1+2\sqrt{x+4}+x+4
\end{matrix}\right.$
$\Leftrightarrow \left\{\begin{matrix}
&x\geq -4 \\
&x-2=\sqrt{x+4}
\end{matrix}\right.$
$\Leftrightarrow \left\{\begin{matrix}
&x\geq 2 \\
&x^{2}-5x=0
\end{matrix}\right.$
$\Leftrightarrow x=0$(không thoả mãn) hoặc $x=5$(thoả mãn)
Chú ý: Các dạng nâng lên luỹ thừa bậc chẵn và lẻ thì làm tương tự như trên, riêng bậc lẻ thì không cần điều kiện.
I. Phương pháp đặt ẩn phụ:
Dạng 1: $(ax+b)^{n}=p.\sqrt[n]{a^{'}x+b^{'}}+qx+r$
+)$p.a^{'}> 0$
Đặt $\sqrt[n]{a^{'}x+b^{'}}=at+b$, sau đó đưa về hệ đối xứng loại 2
+)$p.a^{'}< 0$
Đặt $\sqrt[n]{a^{'}x+b^{'}}=-(at+b)$, sau đó đưa về hệ đối xứng loại 2
VD:$4x^{2}+\sqrt{3x+1}+5=13x$(1)
Đk: $x\geq \dfrac{-1}{3}$
(1)$\Leftrightarrow (2x-3)^{2}=-\sqrt{3x+1}+x+4$
Đặt $\sqrt{3x+1}=-(2y-3)(y\leq \dfrac{3}{2})$
Ta được hệ:$\left\{\begin{matrix}
&(2x-3)^{2}=2y+x+1 \\
&(2y-3)^{2}=3x+1
\end{matrix}\right.$
Trừ 2 phương trình trên vế theo vế ta có:
$4(x^{2}-y^{2})-12(x-y)=2(y-x)$
$\Leftrightarrow (x-y)(2x+2y-5)=0$
+) $x=y\Rightarrow (2x-3)^{2}=3x+1$
$\Leftrightarrow x=\dfrac{15+\sqrt{97}}{8}$(không thoả mãn) hoặc $x=\dfrac{15-\sqrt{97}}{8}$(thoả mãn)
+) $2y=5-2x\Rightarrow (2x-3)^{2}=5-2x+x+1$
$\Leftrightarrow 4x^{2}-11x+3=0$
$\Leftrightarrow x=\dfrac{11-\sqrt{73}}{8}$(không thoả mãn) hoặc $x=\dfrac{11+\sqrt{73}}{8}$(thoả mãn)
Dạng 2: $\alpha .P(x)+\beta .Q(x)=\gamma .\sqrt{P(x).Q(x)}$
+)$P(x)=0\Rightarrow$ Thay vào phương trình
+)$P(x)\neq 0$, ta được: $\alpha +\beta .\dfrac{Q(x)}{P(x)}=\gamma .\sqrt{\dfrac{P(x)}{Q(x)}}$
VD: Giải phương trình: $2x^{2}+5x-1=7\sqrt{x^{3}-1}(2)$
Đk: $x\geq 1$
(2)$\Leftrightarrow 3(x-1)+2(x^{2}+x+1)=7\sqrt{(x-1)(x^{2}+x+1)}$
$\Leftrightarrow 3.\dfrac{x-1}{x^{2}+x+1}+2=7\sqrt{\dfrac{x-1}{x^{2}+x+1}}$
Đặt $\sqrt{\dfrac{x-1}{x^{2}+x+1}}=t\geq 0$
Khi đó ta có: $3t^{2}-7t+2=0$
$\Leftrightarrow t=2$ hoặc $t=\dfrac{1}{3}$
+)$t=2\Rightarrow \sqrt{x-1}=2\sqrt{x^{2}+x+1}$
$\Rightarrow 4x^{2}+3x+5=0$(vô nghiệm)
+)$t=\frac{1}{3}\Rightarrow 3\sqrt{x-1}=\sqrt{x^{2}+x+1}$
$\Rightarrow x^{2}-8x+10=0$
$\Leftrightarrow x=4+\sqrt{6}$(thoả mãn) hoặc $x=4-\sqrt{6}$(thoả mãn)
Dạng 3: $\alpha (P(x)+Q(x))+\beta (\sqrt{P(x)}+\sqrt{Q(x)})\pm 2\alpha \sqrt{P(x)+Q(x)}+\gamma =0$
(trong đó $\alpha ,\beta ,\gamma \in \mathbb{R}$ và $\alpha ^{2}+\beta ^{2}\neq 0$)
Đặt $t=\sqrt{P(x)}\pm \sqrt{Q(x)}$, ta được phương trình: $At^{2}+Bt+C=0$
VD: Giải phương trình: $\sqrt{x-2}-\sqrt{x+2}=2\sqrt{x^{2}-4}-2x+2$(2)
Đk: $x\geq 2$
(2)$\Leftrightarrow \sqrt{x+2}-\sqrt{x-2}=2x-2-2\sqrt{x^{2}-4}$
Đặt $\sqrt{x+2}-\sqrt{x-2}=t\geq 0$
$\Rightarrow t^{2}=2x-2\sqrt{x^{2}-4}$
$\Rightarrow t=t^{2}-2$
$\Leftrightarrow t=-1$(không thoả mãn) hoặc $t=2$(thoả mãn)
$t=2\Rightarrow \sqrt{x+2}=2+\sqrt{x-2}
\Rightarrow x+2=x+2+4\sqrt{x-2}$
$\Leftrightarrow x=2$(thoả mãn)
Dạng 4: $ax^{2}+bx+c=\sqrt{px^{2}+qx+r}$
+)$\dfrac{a}{p}=\dfrac{b}{q}$, đặt $t=\sqrt{px^{2}+qx+r}$, đưa về phương trình bậc 2: $At^{2}+Bt+C=0$
+)$\left\{\begin{matrix}
&p=-b  & \\
&q=\dfrac{1-b^{2}}{a}  & \\
&r=\dfrac{-c(1+b)}{a}  &
\end{matrix}\right.$
Đặt $t=ax^{2}+bx+c$, ta được hệ phương trình đối xứng loại 2
VD: Giải phương trình: $x^{2}+6x-14=\sqrt{98-35x-6x^{2}}$
Đk: $98-35x-6x^{2}\geq 0$
Đặt $x^{2}+6x-14=t\geq 0$
Ta có: $\left\{\begin{matrix}
&x^{2}+6x-14=t \\
&t^{2}+6t-14=x
\end{matrix}\right.$
Trừ 2 phương trình trên vế theo vế ta có:
$t-x=x^{2}-t^{2}+6(x-t)$
$\Leftrightarrow (x-t)(x+t+7)=0$
+)$x=t\Rightarrow x^{2}+6x-14=x$
$\Leftrightarrow x=-7$(không thoả mãn) hoặc $x=2$(thoả mãn)
+)$x+t=-7\Rightarrow x^{2}+7x-7=0$
$\Leftrightarrow x=\dfrac{-7+\sqrt{77}}{2}$(không thoả mãn) hoặc $x=\dfrac{-7-\sqrt{77}}{2}$(không thoả mãn)
II. Phương pháp nhân liên hợp:
Sử dụng: $(\sqrt{a}-\sqrt{b})(\sqrt{a}+\sqrt{b})=a-b$
             $(\sqrt[3]{a}\mp \sqrt[3]{b})(\sqrt[3]{a^{2}}\pm \sqrt[3]{ab}+\sqrt[3]{b^{2}})=a\mp b$
*  Phương pháp sử dụng bất đẳng thức:
Một số bất đẳng thức căn bản:
   - $|A|=|-A| \geq 0$. Dấu “$=$” xảy ra $\Leftrightarrow A=0$
   - $|A| \geq A$. Dấu bằng xảy ra khi $\Leftrightarrow A \geq 0$
   - $a^{2}\geq 0\forall a$. Dấu "=" có khi: $a=0$
   - $|a|\geq a\forall a$. Dấu "=" có khi: $a\geq 0$
   - $|a|+|b|\geq |a+b|$. Dấu "=" có khi: $ab\geq 0$
   - $|a|-|b|\leq |a-b|$. Dấu "=" có khi: $\left\{\begin{matrix}ab\geq 0 & & \\ |a|\geq |b| & & \end{matrix}\right.$
   - $a^{2}+b^{2}\geq 2ab$. Dấu "=" có khi: $a=b$
   - $(a+b)^{2}\geq 4ab\Leftrightarrow ab\leq (\dfrac{(a+b)}{2})^{2}$. Dấu "=" có khi: $a=-b$
   - $\dfrac{1}{a}+\dfrac{1}{b}\geq \frac{4}{a+b}(a;b> 0)$. Dấu "=" có khi: $a=b$
   - $\dfrac{a}{b}+\dfrac{b}{a}\geq 2(ab> 0)$. Dấu "=" có khi: $a=b$
Bất đẳng thức Cô-si (AM-GM):
   Với $n$ số thực dương: $a_{1};a_{2};...;a_{n}$ ta luôn có $\dfrac{a_{1}+a_{2}+...+a_{n}}{n}\geq \sqrt[n]{a_{1}a_{2}...a_{n}}$.
   Dấu "=" khi và chỉ khi: $a_{1}=a_{2}=...=a_{n}$
Bất đẳng thức BCS (Bunhiakovsky):
   Với 2 bộ số thực bất kì: ($a_{1};a_{2};...;a_{n}$);($b_{1};b_{2};...;b_{n}$) ta luôn có:
          $(a_{1}b_{1}+a_{2}b_{2}+...+a_{n}b_{n})^{2}\leq (a_{1}^{2}+a_{2}^{2}+...+a_{n}^{2})(b_{1}^{2}+b_{2}^{2}+...+b_{n}^{2})$
   Dấu "=" có khi: $\dfrac{a_{1}}{b_{1}}=\dfrac{a_{2}}{b_{2}}=...=\dfrac{a_{n}}{b_{n}}$
Bất đẳng thức Svac-xo
   Với $\forall x_{i}>0;i=\overline{1,n}$ ta có: $\dfrac{a_{1}^{2}}{x_{1}}+\dfrac{a_{2}^{2}}{x_{2}}+...+\dfrac{a_{n}^{2}}{x_{n}}\geq \dfrac{(a_{1}+a_{2}+...+a_{n})^{2}}{x_{1}+x_{2}+...+x_{n}}$
Bất đẳng thức Minkopsky:
   Cho 2 dãy số thực dương: $(a_{1};a_{2};...;a_{n});(b_{1};b_{2};...;b_{n})$ ta có:
   $\sqrt{a_{1}^{2}+b_{1}^{2}}+\sqrt{a_{2}^{2}+b_{2}^{2}}+...+\sqrt{a_{n}^{2}+b_{n}^{2}}\geq \sqrt{(a_{1}+a_{2}+...+a_{n})^{2}+(b_{1}+b_{2}+...+b_{n})^{2}}$
   Dấu "=" xảy ra khi: $\dfrac{a_{1}}{b_{1}}=\dfrac{a_{2}}{b_{2}}=...=\dfrac{a_{n}}{b_{n}}$.
...
Trên đây là một số phương pháp mình nêu ra để mọi người cùng tham khảo.